SlideShare a Scribd company logo
1 of 12
PROFESSIONAL EDUCATION
MULTIPLE CHOICE
1. You are very much interested in a quality professional development program for teachers. What characteristic should you
look for?
a. Prescribe by top educational teachers
b. Dependent on the availability of funds
c. Required for renewal of professional license
d. Responsive to identified teacher’s needs.
2. To ensure high standards of teachers’ personal and professional development, which of the following measures must be
implemented?
I. A school head plans the professional development of his/her teachers.
II. Every teacher formulates his/her own professional development plan
III. The implementation of what is learned in a training must be monitored.
a. I only II and III
b. I and III d. II only
3. As a community leader, which of the following should a teacher NOT do?
a. Support effort of the community to improve their status in life.
b. Make herself aloof to ensure that her decisions will not be influenced by the community politics.
c. Solicit donation from philanthropists in the community.
d. Play an active part in the activities of the community.
4. In a highly pluralistic society, what type of learning environment is the responsibility of the teacher?
I. Safe
II. Gender-biased
III. Secure
a. I and II c. II only
b. I, II and III d. I and III
5. A teacher is said to be “trustee of the cultural and educational heritage of the nation and is under obligation to transmit to
learners such heritage”. Which practice makes the teacher fulfill such obligation?
a. Use interactive teaching strategies.
b. Use the latest educational technology.
c. Observe continuing professional education
d. As a class, study the life of Filipino heroes.
6. Which actions show that political factors affect schools as agents of change?
I. The strengthening of the teaching of English in Philippines school.
II. The introduction of mandated subjects such as Rizal in the curriculum
III. The practice of mainstreaming
IV. The turnover of day care centers for DSWD to DepEd for supervision.
a. I and III c. II and III
b. I and II d. II and IV
7. For more efficient and effective management of school as agents of change, one proposal is for the DepEd to cluster remote
stand-alone schools under one lead school head. Which factor has the strongest influence on this proposal?
a. Psychological c. Geographical
b. Historical d. Social
8. What does the acronym EFA imply for schools?
a. The acceptance of exclusive schools for boys and for girls.
b. The stress on the superiority of formal education over that of alternative learning system.
c. Practice of inclusive education
d. The concentration on formal education system
9. The wide acceptance of “bottom up” management style has influenced schools to practice which management practice?
a. Exclusion of politicians from the pool of guest speakers during graduation exercises.
b. Prescription of what ought to be done from the Center Office.
c. Involvement of students, parents, teachers, and community in school planning
d. Allowing schools to do what they think is best
10. Large class size in congested cities is a common problem in our public schools. Which measure/s have schools taken to
offset the effects of large class?
I. The deployment of more teachers
II. The implementation of 1:1 pupil textbook ratio
III. The conduct of morning and afternoon sessions
a. I, II and III c. III only
b. I and II d. II only
11. The failure of independent study with most Filipino students may be attributed to students’
a. unpreparedness fro schooling
b. ambivalence
c. high degree of independence
d. high degree of independence on authority
12. The schooling incidents in school campuses abroad have made school to rethink the curriculum. Which is believed to
counteract such incidents and so is being introduced in schools?
I. Inclusion of socio-emotional teaching
II. The emphasis on the concept of competition against self and not against others
III. Focus on academic achievement and productivity
a. I and III c. I and II
b. II and III d. I, II and III
13. Widespread abuse of Mother Earth prompted schools to teach sustainable development. which one does this prove about
schools?
a. The curricula of schools are centered on Mother Earth.
b. Schools can easily integrate sustained development in their curriculum.
c. Sustained development cannot be effectively taught in the classroom.
d. Environment factors influence the school as an agent of change.
14. A father tells his daughter “You are a woman. You are meant for the home and so for you, going to school is not
necessary.” Is the father CORRECT?
a. It depends on the place when the daughter and the father live.
b. Yes, women are meant to be a mother only.
c. No. today women can take on the jobs of men.
d. No, there is gender equality in education.
15. Is there a legal basis for increasing the teacher’s starting salary to P18,000 an months?
a. No, it is a gift to teachers from Congress
b. Yes, R.A 7836
c. No, it is simply an act of benevolence from President GMA
d. Yes, the Phil. Constitution
16. As provided for the Educational Act of 1982, how are the institutions of learning encourage to set higher standards of
equality over the minimum standards required for state recognition?
a. Granting of Special Permit
b. Academic freedom
c. Continuing Professional Education
d. Voluntary accreditation
17. Despite of opposition from some school official, DepEd has continuously enforced the “no collection of fees” policy during
enrolment period in public schools. In this policy in accordance with EFA goals?
a. No, it violates the mandate of equality education
b. Yes, it somewhat eliminates gender disparities
c. Yes, it supports equitable access to basic education
d. No. it does not support parent of adult education
18. “Specialization is knowing more and more about less and less”. Hence, it is better to be a generalist, claims Teacher F.
Which Philosophy does Teacher F. subscribe to?
a. Existentialism c. Essentialism
b. Perennialism d. Progressivism
19. Mencius believed that all people are born good. This thought on the innate goodness of people makes it easier to
our pupils.
a. teach c. like
b. Respect d. motivate
20. The specialization requires of every professional teacher for him/her to be competent is in line with which pillar of learning?
a. Learning to know
b. Learning to be
c. Learning to live together
d. Learning to do
21. Which pillar of learning is aimed at the wholistic development of man and his complete fulfillment?
a. Learning to be
b. Learning to know
c. Learning to live together
d. Learning to do
22. Material development at the expense of human development points to the need to do more in school.
a. “Learning to do”
b. “Learning to know”
c. “Learning to live together”
d. “Learning to be”
23. A student complains to you about his failing grade. When you recomputed you found out that you committed an error in
this grade computation. Your decision is not accept the erroneous computation before the student and so leave the failing
grade as is for fear that you may lose credibility. Is this morally right?
a. No, the reason for not accepting the error before the students in flimsy.
b. No, the end does not justify the means
c. Yes, the end justifies the means
d. Yes, as a teacher you must maintain your credibility
24. Which violate(s) the principle of respect?
I. Teacher A tells her students that what Teacher B taught is wrong.
II. To retaliate, Teacher B advises students not to enroll in Teacher A’s class.
III. Teacher C secretly gives way to a special favor (e.g. add 2 points to grade) requested by student A who is vying for
honors.
a. II and III c. I and II
b. I, II and III d. I and III
25. Which is/are in accordance with the principle of pedagogical competence?
I. Communication of objectives of the course to students
II. Awareness of alternative instruction strategies
III. Selection of appropriate methods of instruction
a. I and III c. III only
b. I, II and III d. II and III
26. With a death threat over his head, Teacher D is directed to pass an undeserving student. Which will a utilitarianist do?
a. Pass the student, why suffer the threat?
b. Don’t pass him; live by your principle of justice. You will get reward, if not in this life, in the next.
c. Pass the student. That will be of use to the student, his parents and you.
d. Don’t pass him. You surely will not like someone to give you a death threat in other to pass.
27. Teacher A knows of the illegal activities of a neighbor but keeps quiet in order not to be involved in any investigation. Which
foundational principle of morality does Teacher A fail to apply?
a. The end does not justify the means
b. The end justifies the means
c. Always do what is right
d. Between two evils, do the lesser evil
28. To earn units for promotion, a teacher pays her fee but does not attend class at all. Does this constitute professional
growth?
a. Not immediately but yes after promotion
b. It depends on the school she is enrolled in
c. No, it is simply earning MA units for promotion
d. Yes, just enrolling in an MA program is already professional growth
29. If a teacher asks more higher-order questions, he has to ask more questions.
a. fact c. convergent
b. close d. concept
30. Misdemeanor has a “ripple effect”. This implies that as a classroom manager, a teacher
a. reinforces positive behavior
b. responds to misbehavior promptly
c. is consistent in her classroom management practice
d. count 1 to 10 before she deals with a misbehaving student
31. Based on Edgar Dale’s “Cone of Experience”, which activity is farthest from the real thing?
a. Watching demo c. Video disc
b. Attending exhibit d. Viewing images
32. The students of Teacher Y scan an electronic encyclopedia, view a film on subject, or look at related topics at the touch of a
button right there in the classroom. Which device/s des teacher Y’s class have?
a. Teaching machines
b. CD
c. Video disc
d. Videotaped lesson
33. Which is an INAPPROPIATE way to manage off-task behavior?
a. Redirect a child’s attention to task and check his progress to make sure he is continuing work
b. Make eye contact to communicate what you wish to communicate
c. Move closer to the child to make him feel your presence
d. Stop your class activity to correct a child who is no longer on task
34. To be an effective classroom manager, a teacher must be friendly but must at the same time be .
a. confident c. analytical
b. businesslike d. buddy-buddy
35. Which software is needed when one wants to perform automatic calculations on numerical data?
a. Database
b. Spreadsheet Program
c. Microsoft Word
d. Microsoft Powerpoint
36. Which of the following questions must be considered in evaluating teacher-made materials?
a. In the material new?
b. Does the material simulate individualism?
c. Is the material expensive?
37. Kounin claims that “with-it-ness” is one of the characteristics of an effective classroom manager. What is one sign of “with-
it-ness”?
a. Giving attention to students who are having difficulty with school work
b. Seeing only a portion of the class but intensively
c. Knowing where instructional materials are kept
d. Aware of what’s happening in all parts of the classroom
38. Which of these is one of the ways by which the internet enables people to browse documents connected by hypertext
links?
a. URL
b. Browser
c. Welcome page
d. World Wide Web
39. Which characteristics must be primary considered as a choice of instructional aids?
a. Stimulate and maintain students interests
b. Suited to the lesson objectives
c. Updated and relevant to Filipino setting
d. New and skillfully made
40. You can exhibit referent power on the first day of school by
a. telling them the importance of good grades
b. giving your students a sense of belongingness and acceptance
c. making them feel you know what you are taking about
d. reminding your students your authority over them again and again
41. I would like to use a model to emphasize particular part. Which of these would be MOST appropriate?
a. Regalia c. Stimulation
b. Audio recording d. Mock up
42. What must a teacher do to ensure orderly transitions between activities?
a. Allow time for the students to socialize in between activities
b. Have the materials ready at the start of the activity
c. Assign fewer exercise to fill the allotted time
d. Wait for students who lag behind
43. The task of setting up routine activities for effective classroom management is a task that a teacher should undertake
a. as soon as the students have adjusted to their schedules
b. on the very first day of school
c. every day at the start of the session
d. every homeroom day
44. Teacher S uses the low-profile classroom control technique most of the time. what does this imply?
a. She is reactive in her disciplinary orientation
b. She manages pupils personalities
c. She reacts severely to a misbehaving student
d. She stops misbehaving without disrupting lesson flow
45. When teacher tries to elicit clarification on a student response or solicits additional information, which of these should be
use?
a. Directing c. Structuring
b. Probing d. Cross examining
46. Which priority criterion should guide a teacher in the choice of instructional devices?
a. Novelty c. Attractiveness
b. Cost d. Appropriateness
47. Which learning activity is MOST appropriate if a teacher’s focus is attitudinal change?
a. Fieldtrip c. Role play
b. Exhibit d. Game
48. Teacher H strives to draw participation of every student into her classroom discussion. Which of these student needs is she
trying to address? The need to .
a. shoe one’s oral abilities to the rest of the class
b. feel significant and be part of a group
c. get everything and be part of a group
d. be creative
49. Instead of teacher giving this comment a student response. “You’re on the wrong track!”, what should be teacher do?
a. Change the question to an easier one
b. Redirect the question by calling another student to recite
c. Probe to redirect the response into a more productive area
d. Pause, ask the question, lecture, then ask the question again
50. If curriculum is designed following the traditional approach, which feature(s) apply(ies)?
I. The aims of the curriculum are set by professionals and experts
II. Interested groups (teachers, students, communities) are assumed to agree with the aims of the curriculum
III. Consensus building in not necessary
a. III only c. I, II
b. I, II, III d. I, III
51. I want my student to develop the ability to look at a problem from various perspectives. Which approach will be MOST fit?
a. Behaviorist approach
b. Computer-based Education
c. Modular approach
d. Cognitive approach
52. One’s approach to teaching is influenced by Howard Gardner’s MI Theory. What is he/she challenged to do?
I. To come up with 9 different ways of approaching lesson to cater to the 9 multiple intelligence
II. To develop all student’s skill in all nine intelligences
III. To provide worthwhile activities that acknowledge individual difference in children
a. I, II and III c. II only
b. II, III d. III only
53. If my approach to my lesson is behaviorist, what features will dominate my lesson?
I. Copying notes III. Lecturing
II. Reasoning IV. Demonstration
a. III, IV c. I, II, III, IV
b. I, III, IV d. II, III, IV
54. You practice inclusive education. Which of these applies to you?
I. You accept every student as full and valued member of the class and school community
II. Your special attention is on learners with specific learning or social needs
III. Your address the needs of the class as a whole within the context of the learners with specific learning or social needs
a. II only c. I only
b. I and II d. I and III
55. School curriculum reflects the world’s economic and political integration and industrialization. What does this point in
curriculum development?
a. The trend towards the classical approach to curriculum development
b. The trend toward the globalization and localization
c. The trend toward participatory curriculum development
d. The shift in the paradigm of curriculum development from a process-oriented to a product-oriented one
56. You choose cooperative learning as a teaching approach. What thought is impressed on your students?
a. Interaction is a must, but not necessarily face to-face interaction
b. Student’s success depends on the success of the group
c. Student’s individuality evaluate how effectively their group worked.
d. The accountability for learning is on the group not on the individual
57. What principle is violated by overusing the chalkboard, as though it is the only education technology available?
a. Isolated use c. Variety
b. Flexibility d. Uniformity
58. Which statement applies a CORRECTLY to Edgar Dale’s ”Cone of Experience”
a. The farther you are from the base, the more direct the learning experience becomes.
b. The farther you are from the bottom, the more direct the learning experience becomes.
c. The closer you are from the base, the more indirect the learning experience becomes
d. The closer you are from the base, the more direct the learning experience becomes
59. “When more senses are stimulated, teaching and learning become more effective.” What is an application of this principle?
a. Appeal to student’s sense of imagination
b. Use multisensory aids
c. Make your students touch the instructional material
d. Use audiovisual aids because the eyes and the eras are the most important senses in learning.
60. Which is a classroom application of the theory of “operant conditioning”?
a. Help student see the connectedness of facts, concepts, and principles
b. Create a classroom atmosphere that elicits relaxation
c. Reinforce a good behavior to increase the likelihood that the learner will repeat the response
d. Make students learn by operating manipulatives
Read the following teacher-student situation
61. TEACHER: Why is the process called photosynthesis?
STUDENT: I don’t know
Which questioning technique should be the teacher use?
a. Clarification c. Prompting
b. Multiple response d. Concept review
62. Here is the test item.
“From the data presented in the table, from generalizations that are supported by the data”.
Under what type of question does this item fall?
a. Convergent c. Application
b. Evaluative d. Divergent
63. I want to teach concepts, patterns and abstractions. Which method will be MOST appropriate?
a. Discovery c. Direct instruction
b. Indirect instruction d. Problem solving
64. Teacher A teaches English as a Second Language. She uses vocabulary cards, fill-in-the-blanks sentences, dialogues,
dictation and writing exercises in teaching a lesson about grocery shopping. Based on this information, which of the
following is a valid conclusion?
a. The teacher wants to make her teaching easier by having les talk
b. The teacher emphasizing reading and writing skills
c. The teacher is teaching in a variety of ways because not all students learn in the same manner
d. The teacher is applying Bloom’s hierarchy of cognitive learning
65. Teacher A an experienced teacher, does daily review of past lessons in order to
a. introduce a new lesson
b. reflect on how he presented the previous lessons
c. provide his pupils with a sense of continuity
d. determine who among his pupils are studying
66. I combined several subject areas in order to focus on a single concept for interdisciplinary teaching. Which strategy did I
use?
a. Reading-writing activity
b. Thematic introduction
c. Unit method
d. Problem-centered learning
67. To teach the democratic process to the pupils, Biag Elementary School decided that the election of class officers shall be
patterned after local elections. There are qualifications set for candidates, limited period for campaign and rules for posting
campaign materials, etc. Which of the following did the school use?
a. Symposium c. Pole playing
b. Simulation d. Philips 66
68. Which are effective methods in teaching student critical reading skills?
I. Interpret editorial
II. Read and interpret three different movie reviews
III. Read a position paper and deduce underlying assumptions of the position papers
a. II and III c. I and II
b. I and III d. I, II and III
69. Here is a test item
“The improvement of basic education should be the top priority of the Philippine government. Defend or
refute this position”.
Under what type of question does this test item fall?
a. Low-level c. Analysis
b. Evaluative d. Convergent
70. When I teach, I often engage in brainstorming. Which do I avoid?
a. Break down barriers
b. Increase creativity
c. Generate many ideas
d. Selectively involves pupils
71. Teacher S teaches a lesson in which students must recognize that ¼ is the same 0.25. They use this relationship to
determine that 0.15 and 0.20 are slightly less than ¼. Which of the following concept/s is/are being taught?
a. Numeration skills
b. Place value of decimals
c. Numeration skills of decimals and relationships between fractions and decimals
d. Relationship between fraction and decimals
72. What is the best way to develop math concept?
a. Solving problems using multiple approaches
b. Solving problems by looking for correct answer
c. Learning math as applied to situations, such as being a tool of science
d. solving problems by applying learned formulas
73. After the reading of a selection in the class, which of these activities can enhance students creativity.
I. Reader’s theater
II. Reading aloud
III. Silent reading
a. I and II c. I only
b. II only d. III only
74. Teacher C, a Reading teacher, advised he class to “read between the lines”. What does she want her pupils to do?
a. Make an educated guess
b. Determine what is meant by what is stated
c. Apply the information read
d. Describe the characters in the story
75. To nurture students’ creativity, which activity should a teacher AVOID?
a. Ask “hat if…” questions
b. Ask divergent thinking questions
c. Emphasize the need to give right answers
d. Be open to “out-of-this-world” ideas
76. Teacher R wants to develop his student’s creativity. Which type of questions will be MOST appropriate?
a. Synthesis questions
b. Fact questions
c. “What if….” questions
d. Analysis questions
77. In my attempt to develop creative thinking skills, I want to test fluency of ideas. Which activity for my students will be
MOST appropriate?
a. Solve this math problem
b. List animals covered with hair in 1 minute
c. Solve this puzzle
d. Compare pictures 1 and 2. Where are the differences?
78. You want your students to answer the questions at the end of a reading lesson. “What did I learn did?”,”What still puzzle
me?”, “What did I enjoy, hate accomplish in the class today”?,”How did I learn from the lesson?”.Which of the following are
you asking them to do?
a. Work on an assignment
b. Make journal entry
c. Work on a drill
d. Apply what they learned
79. After reading an essay. Teacher B wants to help sharpen her students’ ability to interpret. Which of these activities will be
most appropriate?
a. Drawing conclusions
b. Making inferences
c. Getting the main idea
d. Listing facts separately from opinion
Read the following then answer the question
80. A man and his son are driving in a car. The car crashes into a tree, killing the father and seriously injury his son. At the
hospital, the boy needs to have surgery. Looking at the boy, the doctor says (telling the truth),”I cannot operate on him. He
is my son. How can this be?
ASWER: The doctor is the boy’s mother.
The above brain twister helps develop critical reading skills. Which activity was used?
a. Comparing
b. Classifying
c. Inferring meaning
d. Looking for cause and effect
81. Research says that mastery experiences increase confidence and willingness to try similar or more challenging tasks as
reading. What does this imply for children’s reading performance?
a. Children who have not mastered the basic skills are more likely to be motivated to read in order to gain mastery over
basic skills.
b. Children who have mastered basic skills are more likely to be less motivated to read because they get fed up with too
much reading.
c. Children who have a high sense of self-confidence are not necessarily those who can read
d. Children who have gained mastery over basic skills are more motivated to read
82. The value that students put on reading is critical to their success. In what way/s can teachers inculcate his value for
reading?
I. Sharing the excitement of read-aloud
II. Showing their passion for reading
III. Being rewarded to demonstrate the value of reading
a. II and III c. I, II and III
b. I and II d. II only
83. Bruner’s theory on intellectual development moves from enactive to iconic and symbolic stages. Applying Bruner’s theory.
How would you teach?
a. Be interactive in approach
b. Begin with the abstract
c. Begin with the concrete
d. do direct instruction
84. A person who has painful experiences at the dentist’s office may become fearful at the mere sight of the dentist’s office
building. Which theory can explain this?
a. Generalization
b. Operant Conditioning
c. Attribution theory
d. Classical conditioning
85. Which is/are the basic assumption/s of behaviorists?
I. The mind of newborn child is a blank state.
II. all behaviors are determined by environmental events
III. The child has a certain degree of freedom not to allow himself to be shaped by his environment.
a. III only c. II only
b. I and II d. I and II
86. If a student is encourage to develop himself to the fullest and must satisfy his hierarchy of needs, the highest needs to
satisfy according to Maslow is .
a. psychological need c. belongingness
b. self-actualization d. safety needs
87. In a Social studies class. Teacher I presents a morally ambiguous situation and asks student what they would do. On whose
theory is Teacher I’s technique based?
a. Bandura c. Kohberg
b. Piaget d. Bruner
88. Teacher F is convinced that whatever a student performs a desired behavior, provide reinforcement and soon the student
learns to perform the behavior on his own. On which principle is Teacher F’s conviction based?
a. Environmentalism c. Cognitivism
b. Behaviorism d. Constructivism
89. Bandura’s social learning theory, states that children often imitate those who
I. have substantial influence over their lives
II. belong their peer group
III. belong to other races
IV. are successful and seem admired
a. IV only c. I and II
b. I and IV d. II and IV
90. According to Erikson, what years are critical for the development of self-confidence?
a. High school years
b. College years
c. Preschool years
d. Elementary school years
91. Which of the following does NOT describe the development of children aged 11 to 13?
a. They exhibit increased objectivity in thinking
b. They shift from impulsivity to adaptive ability
c. Sex difference in IQ become more evident
d. They show abstract thinking and judgment
92. Teacher H begins a lesson on tumbling, demonstrating front and back somersaults in slow motion and physically guiding his
students through the correct movements. As his students become more skillful, he stands back from the man and gives
verbal feedback about how to improve. With Vygotsky’s theory in mind, what did Teacher H do?
a. Apprenticeship
b. Guided participation
c. Peer interaction
d. Scaffolding
93. What does Gagne’s hierarchy theory propose for effective instruction?
a. Be concerned with the socio-emotional climate in the classroom
b. Teach beginning with the concrete
c. Sequence instruction
d. Reward good behavior
94. William Glasser’s control theory states that behavior in inspired by what satisfies a person’s want at any given time. What
then must a teacher do to motivate students to learn?
a. Make teaching-leaning interactive
b. Avoid giving assignments
c. Organize a curriculum in a spiral manner
d. Make schoolwork relevant to students’ basic human needs.
95. Soc exhibit fear response to freely roaming dogs but does not show fear when a dog is on a leash or confined to a pen.
Which conditioning process is illustrated?
a. extinction c. acquisition
b. generalization d. discrimination
96. Based on Freud’s theory, which operate/s when a student strikes a classmates at the height of anger?
a. Ego c. Id and Ego interact
b. Id d. Superego
97. Bernadette enjoyed the roller coaster when he and her family went to Enchanted Kingdom. The mere sight of a roller
coaster gets her excited. Which theory explains Bernadette’s behavior?
a. Operant conditioning
b. Social learning theory
c. Attribution theory
d. Pavlovian conditioning
98. According to Frued, with which should one be concerned if he/she has to develop in the students a correct sense of right
and wrong?
I. Super-ego II. Ego III. Id
a. I and II c. I
b. II d. III
99. When small children call animals “dog”, what process is illustrated on Piaget’s cognitive development theory?
a. reversion c. accommodation
b. assimilation d. conservation
100. Researchers found that when a child is engaged in a learning experience a number of areas of the brain are
simultaneously activated. Which of the following is/are implication/s of this research finding?
I. Make use of field trips, guest speakers
II. Do multicultural units of study
III. Stick to the “left brain and right brain” approach
a. I and III c. I and II
b. I only d. II only
101. Which appropriate teaching practice flows from this research finding on the brain: “The brain’s emotional center is tied
into its ability to learn”.
a. Establish the discipline of being judgmental in attitude
b. Come up with highly competitive games where winners will feel happy
c. Tell the students to participate in class activities or else won’t receive plus points in class recitation
d. Create a learning environment that encourages students to explore their feeling and ideas freely
102. Research on Piagetian tasks indicates that thinking becomes more logical and abstract as children reach the formal
operations stage. What is an educational implication of this finding?
a. Engage children in analogical reasoning as early as preschool to train them for higher order thinking skills (HOTS)
b. Learners who are not capable of logical reasoning from ages 8 to 11 lag behind in their cognitive development
c. Let children be children
d. Expect hypothetical reasoning for learners between 12 to 15 years of age
103. Research says: “People tend to attribute their successes to internal causes and their failures to external causes.”Based
on this finding, what should be taught to students for them to be genuinely motivated to succeed?
a. Tell them the research finding when applied will make them genuinely motivated
b. Convince them that genuine motivation is the only factor that matters for a person to succeed
c. Make them realize that failure is a part of life
d. Make them realize that both success and failure are more a function of internal causes.
104. Which characterize/s a learning environment that promotes fairness among learners of various cultures, family
background and gender?
I. Inclusive
II. Exclusive
III. Gender-sensitive
a. I only c. I and III
b. III only d. II and III
105. Which of the following steps should be completed first in planning an achievement test?
a. Define the instructional objective
b. Set up a table of specialization
c. Select the types of test items to use
d. Decide on the length of the test
106. The computed r fro scores in Math and Science in 0.92. What does this mean?
a. Math score is positive related to Science score
b. The higher the Math score, the lower the Science score
c. Math score is not in any way related to Science score
d. Science score is slightly related to math score
107. Which types of test is most appropriate if Teacher Y wants to measure student’s ability to organize thoughts and ideas?
a. Short answer type of test
b. Extended response essay
c. Modified alternative response
d. Limited response essay
108. With assessment of affective learning in mind, which does NOT belong to the group?
a. Cloze test c. Reflective writing
b. Moral dilemma d. Diary entry
109. I want to test student’s synthesizing skills. Which has the highest diagnostic value?
a. multiple choice test c. Essay test
b. Performance test d. Completion test
110.
Why is this test item poor?
is an example of a leafy
vegetable.
I. The test item does not pose a problem to the examinee
II. There are variety of possible correct answer to this item
III. the language used in the question is not precise
IV. The blank is near the beginning of a sentence
A. I and III
B. II and IV
C. I and IV
D. I and II
111.
What makes the multiple choice type of test poor?
a. The options are not grammatically connected to the stem
b. The stem fails to present a problem
c. There are grammatical clues
d. The options are not parallel
112. If a teacher wants to measure her students’ ability to discriminate, which of these is an appropriate type of test item as
implied by the direction?
a. “Outline the Chapter on The Cell”.
b. “Summarize the lesson yesterday”.
c. “Group the following items according to shape”.
d. “State a set of principle that can explain the following events”.
113. A test item has a difficult index of 0.89 and a discrimination index of 0.44. What should the teacher do?
a. Reject the item
b. Retain the item
c. Make it a bonus item
d. Make it a bonus item and reject it
114. Which form of assessment is consistent with the saying “The proof of the pudding is in the eating”.
a. Contrived c. Traditional
b. Authentic d. Indirect
115.
What is WRONG with this item?
a. Item is overly specific
b. Content is trivial
c. Test item is option-based
d. There is a cue to the right answer
116. Student’s score were as follows: 82, 83, 84, 86, 88, 84, 83, 85. The score 86 is the.
a. mode c. median
b. average d. mean
117. Which text form would you choose if you want to have a valid and reliable test based on the table below?
Test Form Validity Index Reliability Index
A .47 .68
B .87 .57
C .20 .86
D .40 .41
E .63 .07
a. A only c. A and D
b. B only d. B and E
118. A mathematicians test was given to all Grade V pupils to determine the contestants for the Math Quiz Bee. Which
statistical measure should be used to identify the top 15?
a. Mean percentage score
b. Quartile Deviation
c. Percentile Rank
d. Percentage Score
119. Nellie’s score is within x±1 SD. To which of the following groups does she belong?
a. Below average
b. Average
c. Needs Improvement
d. Above average
120. Use the inbox below to answer the question that follows:
Percentage Grades for Final Examination
What follows is a multiple choice type of
test.
Some test items .
a. are too difficult
b. are objective
c. are poorly constructed
d. have multiple defensible answers
Who is best admired for outstanding
contribution to world peace?
a. Kissinger c. Kennedy
b. Clinton d. Mother Teresa
40 70 80 90 100
Which of the following statement is TRUE about the plot of grades above?
a. The median is a score of 80 and the range is 60.
b. The median is a score of 70 and the range is 60.
c. The median is a score of 80 and the range is 20.
d. The median is a score of 70 and the range is 20.
121. Which can be said of Arielle who obtained a score of 75 out of 100 items in a Grammar objective test?
a. She performed better than 25% of her classmates
b. She answered 75 items in the test correctly
c. Her rating is 75
d. She answered 75% of the test items correctly
122. The criterion of success in Teacher D’s objective is that “the pupils must be able to spell 90% of the words correctly”.
Ana and 24 others in the class spelled only 40 out of 50 words correctly while the rest scored 45 and above. This means
that Teacher D
a. attained her lesson objective
b. did not attain her lesson objective because of the pupils’ lack of attention
c. failed to attain her lesson objective as far as the 25 pupils are concerned
d. attained her lesson objective because of her effective spelling drill
123. If the scores of your test follow a negatively skewed score distribution, what should you do?
Find out ?
a. why your items were easy
b. why most of the scores are high
c. why most of the scores are low
d. why some pupils scored high
124. Principal A is talking about “grading on the curve” in a faculty meeting. What does this expression refer to?
a. A student mark compares his achievement to his effort.
b. A student’s grade or mark depends on how his achievement compares with the achievement of other students in a
class.
c. A student’s grade determines whether or not a student attains a defined standard of achievement
d. A student mark tells how closely he is achieving to his potential.
125. Which tests determine whether students accept responsibility for their own behavior or pass on responsibility for their
own behavior to other people?
a. Thematic tests
b. Sentence-completion tests
c. Stylistic test
d. Locus-of-control tests
126. Which of the one weakness of self-supporting personality checklists?
a. Many personality measures have built-in lie scales
b. They lack stability
c. They may not get true information because individuals can hide or disguise feelings
d. They have poor internal consistency
127. Which of these can measure awareness of values?
a. Sociogram
b. Moral dilemmas
c. Projective techniques
d. Rating scales
128. Marking on a normative basis means that
a. the normal distribution curve should be followed
b. some should fall
c. some get high marks
d. the grading is based on a present criteria
129. Which process enhances the comparability of grades?
a. Using a table specifications
b. Determining the level of difficulty of the tests
c. Giving more HOTS (higher order thinking skills)
d. Constructing departmentalized examinations for each subject area.
SITUATIONAL
130. Which response/s come/s from a behaviorist?
a. #2 and #4 c. #3 and #4
b. #1 and #2 d. #1 and #3
131. On which educational philosophy is response #1 anchored?
a. Existentialism c. Progressivism
b. Essentialism d. Bahaviorism
132. If you learned toward a progressivist philosophy, with which response would you agree?
a. #2 b. #3 c. #4 d. #1
Situation 1- In a faculty meeting, the
principle told his teacher: “We need to
improve our school performance in the
National Achievement Test. What should
we do?
The teacher gave varied answers as
follows:
1. Let’s give incentives and rewards to
students who get a rating of 85%
2. Let’s teach them to accept complete
responsibility for their performance
3. Let’s make the school environment
conducive for learning
4. Let’s make use of the experimental
methods of teaching
Situation 2- One principle in the utilization of
technology of the classroom is appropriateness
of material or activity.
133. Teacher C wants his students to master the concept of social justice. Which series of activities will be most effective?
a. Pretest-teaching-posttest
b. Pretest-teaching-posttest-re-teaching for unlearned concepts-posttest
c. Review-pretest-teaching-posttest
d. Teaching-posttest
134. Teacher A likes to show how the launching of spaceships takes place. Which of the following materials available is most
fit?
a. Model c. Replica
b. Mock-up d. Realia
135. Teacher B likes to concretize the abstract concepts of an atom. She came up with a concrete presentation of the atom
by using wire and plastic balls. How would you classify Teacher B’s visual aids?
a. Chart c. Model
b. Replica d. Realia
136. The class was asked to share their insights about the poem. The ability to come up with a n insight stems from the
ability to
a. analyze the parts of a whole
b. evaluate the worthiness of a thing
c. relate and organize things and ideas
d. comprehend the subject that is being studied
137. To ask the class any insight derived from the poem is based on the theory of
a. realism c. conditioning
b. behaviorism d. constructivism
138. On which assumption about the learner is Mr. Marquez’s act of asking the class to share their insight based?
a. Learners are like empty receptacles waiting to be filled up
b. Learners are meant to interact with one another
c. Learners have multiple intelligence and varied learning styles
d. Learners are producers of knowledge not only passive recipients of information
139. On which assumption/s is the principal’s action anchored?
I. Students learn by personally constructing meaning of what is taught.
II. Students are construct and reconstruct meaning based on experiences
III. Students derive meaning from the meaning that the teacher gives
a. II only c. I, II, and III
b. I and II d. I only
140. Which materials will her teachers LEAST prefers?
a. Controversial issues
b. Open-ended topics
c. Unquestionable laws
d. Problem or cases
141. Which concept/s of the learner will Principal E NOT accept?
I. “Empty vesse!”
II. “Tabula rasa”
III. Candle to be lighted
a. III only c. II only
b. I only d. I and II
142. How can you make the items homogeneous?
a. Increase the number of items in Column B
b. All items should be on polygons
c. Remove the word triangle in items #1 and #2 in column A
d. The word “gon” must be included in column B
143. What is the main defect of this matching test?
a. the matching type is an imperfect type
b. the items are NOT homogeneous
c. the items quite easy
d. an obvious pattern is followed in the answering
144. Which should be done to improve the matching type of test?
a. Capitalize the items in Column A
b. Items in Column A and B should be exchanged
c. Drop #6 item in Column A
Situation 3- After reading and paraphrasing
Robert frost’s “Stopping by the Wood on a
snowy Evening”. Mr. Sales asked the class to
Situation 4- Principal E wants her teachers
to apply constructivism in teaching
Situation 5- Study the matching type of test then
answer the 3 questions that follow:
Column A Column B
1. equilateral triangle A. With 3 equal sides
2. right triangle B. With 5 equal sides
3. octagon C. Has 90- degree
angle
4. pentagon D. Means many
5. heptagon E. with 7 sides
6. poly F. with 8 sides
d. The item in Column A should be increased
145. Which of these is/are essential in constructing a scoring rubric?
I. Description of criteria to serve as standard
II. Clear descriptions of performance at each level
III. Levels of achievement (mastery)
IV. Rating scheme
a. I, II, III c. I, II, III, IV
b. I, II d. I only
146. Which statement is TRUE of the rubric?
a. It is developmental
b. It is analytical
c. It is both holistic and developmental
d. It is holistic
147. Which is TRUE of the scoring rubric?
I. It describes criteria of levels of achievement
II. It has a rating scheme
III. It limit itself to 4 levels of achievement
a. I and II
b. I and III
c. II and III
d. I, II and III
148. The table shows that the test item analyzed .
a. has a positive discrimination index
b. has a negative discrimination index
c. is extremely easy
d. is extremely difficult
149. Based on the table, which is the most effective distracter?
a. Option D
b. Option A
c. Option C
d. Option B
150. Based on the table, which group got more correct answer?
a. Upper group
b. It cannot be determined
c. Lower group
d. Data are not sufficient to give an answer
When you get right down to the root of the meaning of the word "succeed," you find that it simply means to follow through.
Situation 6- Below the template for Scoring
Rubric.
5-Demonstrate complete understanding of the
problem. All requirements of task are included in
response
4-Demonstrate considerable understanding of
the problem. All requirements of task are
included
3- Demonstrate partial understanding of the
problem. Most requirements of task are included
2- Demonstrate little understanding of the
problem. Many requirements of task are missing
1- Demonstrate no understanding of the problem
0-No response/task not attempted
Situation 7- Study the table on item analysis for
non-attractiveness and non-plausibility of
distracters based on the results of a try-out test
in Science. The letter marked with a asterisk is
the correct answer.
Item No. 1 A B C D E
Upper 27% 10 4 1 1 0
Lower 27% 6 5 2 2 0
Answer Key for prof ed
1 C 51 D 101 D
2 B 52 B 102 D
3 C 53 B 103 C
4 D 54 D 104 C
5 D 55 D 105 A
6 D 56 C 106 A
7 C 57 C 107 B
8 C 58 D 108 A
9 C 59 B 109 B
10 A 60 C 110 D
11 D 61 A 111 B
12 D 62 B 112 C
13 B 63 A 113 B
14 D 64 D 114 B
15 D 65 C 115 D
16 C 66 B 116 D
17 C 67 B 117 A
18 C 68 D 118 C
19 A 69 C 119 D
20 A 70 D 120 A
21 A 71 D 121 B
22 A 72 C 122 C
23 B 73 C 123 B
24 B 74 B 124 B
25 B 75 C 125 D
26 C 76 C 126 C
27 C 77 B 127 B
28 C 78 C 128 D
29 D 79 B 129 D
30 B 80 D 130 B
31 D 81 D 131 D
32 A 82 C 132 B
33 D 83 A 133 B
34 B 84 D 134 D
35 B 85 C 135 B
36 C 86 B 136 A
37 D 87 C 137 D
38 A 88 D 138 B
39 B 89 C 139 B
40 C 90 D 140 C
41 D 91 C 141 A
42 B 92 B 142 B
43 C 93 C 143 B
44 A 94 D 144 C
45 B 95 D 145 C
46 D 96 B 146 C
47 C 97 A 147 A
48 B 98 C 148 D
49 B 99 D 149 D
50 B 100 C 150 C

More Related Content

What's hot

FS 1 Episodes 1-6 plus attachments
FS 1 Episodes 1-6 plus attachmentsFS 1 Episodes 1-6 plus attachments
FS 1 Episodes 1-6 plus attachmentsMarian Tiempo
 
Professional education
Professional educationProfessional education
Professional educationAlex Acayen
 
Detailed lesson plan in math 3 final
Detailed lesson plan in math 3 finalDetailed lesson plan in math 3 final
Detailed lesson plan in math 3 finalMyraDelosSantos5
 
Cybersafety Version1
Cybersafety Version1Cybersafety Version1
Cybersafety Version1sabrinacotta
 
MAPEH arts lesson plan for grade 1-2
MAPEH arts lesson plan for grade 1-2MAPEH arts lesson plan for grade 1-2
MAPEH arts lesson plan for grade 1-2Via Martinez Abayon
 
Educ 1 Learning Module COMPLETED.pdf
Educ 1 Learning Module COMPLETED.pdfEduc 1 Learning Module COMPLETED.pdf
Educ 1 Learning Module COMPLETED.pdfRichardAbarracoso
 
Lesson Plan in Math II
Lesson Plan in Math IILesson Plan in Math II
Lesson Plan in Math IIjanehbasto
 
Fs 1 episode 3 classroom management and learning
Fs 1 episode 3 classroom management and learningFs 1 episode 3 classroom management and learning
Fs 1 episode 3 classroom management and learningNoel Parohinog
 
Factors for Technology Selection
Factors for Technology SelectionFactors for Technology Selection
Factors for Technology SelectionAlbin Caibog
 
GLOBAL EDUCATION AND GLOBAL TEACHER
GLOBAL EDUCATION AND GLOBAL TEACHERGLOBAL EDUCATION AND GLOBAL TEACHER
GLOBAL EDUCATION AND GLOBAL TEACHEREduard Orsal
 
Brief lesson plan - Gerund
Brief lesson plan - GerundBrief lesson plan - Gerund
Brief lesson plan - GerundAubrey Rose
 
4A's Detailed Lesson Plan about Basic Exercises
4A's Detailed Lesson Plan about Basic Exercises 4A's Detailed Lesson Plan about Basic Exercises
4A's Detailed Lesson Plan about Basic Exercises Rexzel Rabacal
 
4A's Lesson Plan in English 6
4A's Lesson Plan in English 64A's Lesson Plan in English 6
4A's Lesson Plan in English 6janehbasto
 
Module 1 - The Teaching Profession.pdf
Module 1 - The Teaching Profession.pdfModule 1 - The Teaching Profession.pdf
Module 1 - The Teaching Profession.pdfTinAdlawan
 
Global Issues That Concern Schools and Society.docx
Global Issues That Concern Schools and Society.docxGlobal Issues That Concern Schools and Society.docx
Global Issues That Concern Schools and Society.docxSheldonDarylToledo
 
Detailed Lesson Plan in Arts for Primary Level
Detailed Lesson Plan in Arts for Primary LevelDetailed Lesson Plan in Arts for Primary Level
Detailed Lesson Plan in Arts for Primary Leveljanehbasto
 

What's hot (20)

FS 1 Episodes 1-6 plus attachments
FS 1 Episodes 1-6 plus attachmentsFS 1 Episodes 1-6 plus attachments
FS 1 Episodes 1-6 plus attachments
 
Professional education
Professional educationProfessional education
Professional education
 
Akon1
Akon1Akon1
Akon1
 
Detailed lesson plan in math 3 final
Detailed lesson plan in math 3 finalDetailed lesson plan in math 3 final
Detailed lesson plan in math 3 final
 
Cybersafety Version1
Cybersafety Version1Cybersafety Version1
Cybersafety Version1
 
MAPEH arts lesson plan for grade 1-2
MAPEH arts lesson plan for grade 1-2MAPEH arts lesson plan for grade 1-2
MAPEH arts lesson plan for grade 1-2
 
Educ 1 Learning Module COMPLETED.pdf
Educ 1 Learning Module COMPLETED.pdfEduc 1 Learning Module COMPLETED.pdf
Educ 1 Learning Module COMPLETED.pdf
 
Lesson Plan in Math II
Lesson Plan in Math IILesson Plan in Math II
Lesson Plan in Math II
 
Fs 1 episode 3 classroom management and learning
Fs 1 episode 3 classroom management and learningFs 1 episode 3 classroom management and learning
Fs 1 episode 3 classroom management and learning
 
Detailed Lesson Plan in Science 5
Detailed Lesson Plan in Science 5Detailed Lesson Plan in Science 5
Detailed Lesson Plan in Science 5
 
Field study 1 episode 4
Field study 1 episode 4Field study 1 episode 4
Field study 1 episode 4
 
Factors for Technology Selection
Factors for Technology SelectionFactors for Technology Selection
Factors for Technology Selection
 
Physical education lesson plan
Physical education lesson planPhysical education lesson plan
Physical education lesson plan
 
GLOBAL EDUCATION AND GLOBAL TEACHER
GLOBAL EDUCATION AND GLOBAL TEACHERGLOBAL EDUCATION AND GLOBAL TEACHER
GLOBAL EDUCATION AND GLOBAL TEACHER
 
Brief lesson plan - Gerund
Brief lesson plan - GerundBrief lesson plan - Gerund
Brief lesson plan - Gerund
 
4A's Detailed Lesson Plan about Basic Exercises
4A's Detailed Lesson Plan about Basic Exercises 4A's Detailed Lesson Plan about Basic Exercises
4A's Detailed Lesson Plan about Basic Exercises
 
4A's Lesson Plan in English 6
4A's Lesson Plan in English 64A's Lesson Plan in English 6
4A's Lesson Plan in English 6
 
Module 1 - The Teaching Profession.pdf
Module 1 - The Teaching Profession.pdfModule 1 - The Teaching Profession.pdf
Module 1 - The Teaching Profession.pdf
 
Global Issues That Concern Schools and Society.docx
Global Issues That Concern Schools and Society.docxGlobal Issues That Concern Schools and Society.docx
Global Issues That Concern Schools and Society.docx
 
Detailed Lesson Plan in Arts for Primary Level
Detailed Lesson Plan in Arts for Primary LevelDetailed Lesson Plan in Arts for Primary Level
Detailed Lesson Plan in Arts for Primary Level
 

Similar to Prof ed set b questionati and answer

Let reviewer 2017
Let reviewer 2017Let reviewer 2017
Let reviewer 2017Arneyo
 
Review profed-part-b-with-answers-1
Review profed-part-b-with-answers-1Review profed-part-b-with-answers-1
Review profed-part-b-with-answers-1John Paul Intano
 
Let professional education 7
Let professional education 7Let professional education 7
Let professional education 7Alex Acayen
 
PROFESSIONAL EDUCATION SECONDARY.docx
PROFESSIONAL EDUCATION SECONDARY.docxPROFESSIONAL EDUCATION SECONDARY.docx
PROFESSIONAL EDUCATION SECONDARY.docxFreeEarningSiteandAp
 
Let professional education 1
Let professional education 1Let professional education 1
Let professional education 1Alex Acayen
 
Let2013 march (1)
Let2013 march (1)Let2013 march (1)
Let2013 march (1)Arneyo
 
Let2013 march (1)
Let2013 march (1)Let2013 march (1)
Let2013 march (1)Arneyo
 
Professional education reviewer for let or blept examinees
Professional education reviewer for let or blept examineesProfessional education reviewer for let or blept examinees
Professional education reviewer for let or blept examineeselio dominglos
 
Qualifying exam for school heads 2018
Qualifying exam for school heads 2018Qualifying exam for school heads 2018
Qualifying exam for school heads 2018Noel Dauran
 
Foundations of-education-prof.ed
Foundations of-education-prof.edFoundations of-education-prof.ed
Foundations of-education-prof.edKendral Flores
 
Reviewer Let professional education 13
Reviewer Let professional education 13Reviewer Let professional education 13
Reviewer Let professional education 13Daniel Bragais
 
Let professional education 2
Let professional education 2Let professional education 2
Let professional education 2Alex Acayen
 
Teaching profession part 1
Teaching profession part 1Teaching profession part 1
Teaching profession part 1chinnex23
 
Prof ed (prc)
Prof ed (prc)Prof ed (prc)
Prof ed (prc)Arneyo
 
PROFESSIONAL EDUCATION 3.pptx
PROFESSIONAL EDUCATION 3.pptxPROFESSIONAL EDUCATION 3.pptx
PROFESSIONAL EDUCATION 3.pptxAljonMendoza3
 

Similar to Prof ed set b questionati and answer (20)

Let reviewer 2017
Let reviewer 2017Let reviewer 2017
Let reviewer 2017
 
Review profed-part-b-with-answers-1
Review profed-part-b-with-answers-1Review profed-part-b-with-answers-1
Review profed-part-b-with-answers-1
 
drill4.6.28.pptx
drill4.6.28.pptxdrill4.6.28.pptx
drill4.6.28.pptx
 
Let professional education 7
Let professional education 7Let professional education 7
Let professional education 7
 
PROFESSIONAL EDUCATION SECONDARY.docx
PROFESSIONAL EDUCATION SECONDARY.docxPROFESSIONAL EDUCATION SECONDARY.docx
PROFESSIONAL EDUCATION SECONDARY.docx
 
Let professional education 1
Let professional education 1Let professional education 1
Let professional education 1
 
Prof ed questions
Prof ed questionsProf ed questions
Prof ed questions
 
ProfEd Drills
ProfEd DrillsProfEd Drills
ProfEd Drills
 
Let2013 march-1
Let2013 march-1Let2013 march-1
Let2013 march-1
 
Let2013 march (1)
Let2013 march (1)Let2013 march (1)
Let2013 march (1)
 
Let2013 march (1)
Let2013 march (1)Let2013 march (1)
Let2013 march (1)
 
Professional education reviewer for let or blept examinees
Professional education reviewer for let or blept examineesProfessional education reviewer for let or blept examinees
Professional education reviewer for let or blept examinees
 
Qualifying exam for school heads 2018
Qualifying exam for school heads 2018Qualifying exam for school heads 2018
Qualifying exam for school heads 2018
 
Foundations of-education-prof.ed
Foundations of-education-prof.edFoundations of-education-prof.ed
Foundations of-education-prof.ed
 
Kunot
KunotKunot
Kunot
 
Reviewer Let professional education 13
Reviewer Let professional education 13Reviewer Let professional education 13
Reviewer Let professional education 13
 
Let professional education 2
Let professional education 2Let professional education 2
Let professional education 2
 
Teaching profession part 1
Teaching profession part 1Teaching profession part 1
Teaching profession part 1
 
Prof ed (prc)
Prof ed (prc)Prof ed (prc)
Prof ed (prc)
 
PROFESSIONAL EDUCATION 3.pptx
PROFESSIONAL EDUCATION 3.pptxPROFESSIONAL EDUCATION 3.pptx
PROFESSIONAL EDUCATION 3.pptx
 

More from Rennie Boy Rosadia

Professional education-test-with-rational-answers1 (1)
Professional education-test-with-rational-answers1 (1)Professional education-test-with-rational-answers1 (1)
Professional education-test-with-rational-answers1 (1)Rennie Boy Rosadia
 
Professional education legal bases for ph education 1 (3)
Professional education legal bases for ph education 1 (3)Professional education legal bases for ph education 1 (3)
Professional education legal bases for ph education 1 (3)Rennie Boy Rosadia
 
Professional education legal bases for ph education 1 (3) (1)
Professional education legal bases for ph education 1 (3) (1)Professional education legal bases for ph education 1 (3) (1)
Professional education legal bases for ph education 1 (3) (1)Rennie Boy Rosadia
 
Preboard secondary general-education (1)
Preboard secondary general-education (1)Preboard secondary general-education (1)
Preboard secondary general-education (1)Rennie Boy Rosadia
 
Met exam-appears-in-gen-ed. (2)
Met exam-appears-in-gen-ed. (2)Met exam-appears-in-gen-ed. (2)
Met exam-appears-in-gen-ed. (2)Rennie Boy Rosadia
 
Legal bases-of-philippine-educational-system
Legal bases-of-philippine-educational-systemLegal bases-of-philippine-educational-system
Legal bases-of-philippine-educational-systemRennie Boy Rosadia
 
General education-with-answers
General education-with-answersGeneral education-with-answers
General education-with-answersRennie Boy Rosadia
 
Curriculum development handouts
Curriculum development handoutsCurriculum development handouts
Curriculum development handoutsRennie Boy Rosadia
 

More from Rennie Boy Rosadia (11)

Let filipino1
Let filipino1Let filipino1
Let filipino1
 
Professional education-test-with-rational-answers1 (1)
Professional education-test-with-rational-answers1 (1)Professional education-test-with-rational-answers1 (1)
Professional education-test-with-rational-answers1 (1)
 
Professional education legal bases for ph education 1 (3)
Professional education legal bases for ph education 1 (3)Professional education legal bases for ph education 1 (3)
Professional education legal bases for ph education 1 (3)
 
Professional education legal bases for ph education 1 (3) (1)
Professional education legal bases for ph education 1 (3) (1)Professional education legal bases for ph education 1 (3) (1)
Professional education legal bases for ph education 1 (3) (1)
 
Prof ed-a
Prof ed-aProf ed-a
Prof ed-a
 
Preboard secondary general-education (1)
Preboard secondary general-education (1)Preboard secondary general-education (1)
Preboard secondary general-education (1)
 
Met exam-appears-in-gen-ed. (2)
Met exam-appears-in-gen-ed. (2)Met exam-appears-in-gen-ed. (2)
Met exam-appears-in-gen-ed. (2)
 
Let filipino1
Let filipino1Let filipino1
Let filipino1
 
Legal bases-of-philippine-educational-system
Legal bases-of-philippine-educational-systemLegal bases-of-philippine-educational-system
Legal bases-of-philippine-educational-system
 
General education-with-answers
General education-with-answersGeneral education-with-answers
General education-with-answers
 
Curriculum development handouts
Curriculum development handoutsCurriculum development handouts
Curriculum development handouts
 

Recently uploaded

Hierarchy of management that covers different levels of management
Hierarchy of management that covers different levels of managementHierarchy of management that covers different levels of management
Hierarchy of management that covers different levels of managementmkooblal
 
Framing an Appropriate Research Question 6b9b26d93da94caf993c038d9efcdedb.pdf
Framing an Appropriate Research Question 6b9b26d93da94caf993c038d9efcdedb.pdfFraming an Appropriate Research Question 6b9b26d93da94caf993c038d9efcdedb.pdf
Framing an Appropriate Research Question 6b9b26d93da94caf993c038d9efcdedb.pdfUjwalaBharambe
 
Gas measurement O2,Co2,& ph) 04/2024.pptx
Gas measurement O2,Co2,& ph) 04/2024.pptxGas measurement O2,Co2,& ph) 04/2024.pptx
Gas measurement O2,Co2,& ph) 04/2024.pptxDr.Ibrahim Hassaan
 
Employee wellbeing at the workplace.pptx
Employee wellbeing at the workplace.pptxEmployee wellbeing at the workplace.pptx
Employee wellbeing at the workplace.pptxNirmalaLoungPoorunde1
 
Like-prefer-love -hate+verb+ing & silent letters & citizenship text.pdf
Like-prefer-love -hate+verb+ing & silent letters & citizenship text.pdfLike-prefer-love -hate+verb+ing & silent letters & citizenship text.pdf
Like-prefer-love -hate+verb+ing & silent letters & citizenship text.pdfMr Bounab Samir
 
Introduction to AI in Higher Education_draft.pptx
Introduction to AI in Higher Education_draft.pptxIntroduction to AI in Higher Education_draft.pptx
Introduction to AI in Higher Education_draft.pptxpboyjonauth
 
How to Configure Email Server in Odoo 17
How to Configure Email Server in Odoo 17How to Configure Email Server in Odoo 17
How to Configure Email Server in Odoo 17Celine George
 
Difference Between Search & Browse Methods in Odoo 17
Difference Between Search & Browse Methods in Odoo 17Difference Between Search & Browse Methods in Odoo 17
Difference Between Search & Browse Methods in Odoo 17Celine George
 
Grade 9 Q4-MELC1-Active and Passive Voice.pptx
Grade 9 Q4-MELC1-Active and Passive Voice.pptxGrade 9 Q4-MELC1-Active and Passive Voice.pptx
Grade 9 Q4-MELC1-Active and Passive Voice.pptxChelloAnnAsuncion2
 
EPANDING THE CONTENT OF AN OUTLINE using notes.pptx
EPANDING THE CONTENT OF AN OUTLINE using notes.pptxEPANDING THE CONTENT OF AN OUTLINE using notes.pptx
EPANDING THE CONTENT OF AN OUTLINE using notes.pptxRaymartEstabillo3
 
Proudly South Africa powerpoint Thorisha.pptx
Proudly South Africa powerpoint Thorisha.pptxProudly South Africa powerpoint Thorisha.pptx
Proudly South Africa powerpoint Thorisha.pptxthorishapillay1
 
ECONOMIC CONTEXT - PAPER 1 Q3: NEWSPAPERS.pptx
ECONOMIC CONTEXT - PAPER 1 Q3: NEWSPAPERS.pptxECONOMIC CONTEXT - PAPER 1 Q3: NEWSPAPERS.pptx
ECONOMIC CONTEXT - PAPER 1 Q3: NEWSPAPERS.pptxiammrhaywood
 
Judging the Relevance and worth of ideas part 2.pptx
Judging the Relevance  and worth of ideas part 2.pptxJudging the Relevance  and worth of ideas part 2.pptx
Judging the Relevance and worth of ideas part 2.pptxSherlyMaeNeri
 
Earth Day Presentation wow hello nice great
Earth Day Presentation wow hello nice greatEarth Day Presentation wow hello nice great
Earth Day Presentation wow hello nice greatYousafMalik24
 
ECONOMIC CONTEXT - LONG FORM TV DRAMA - PPT
ECONOMIC CONTEXT - LONG FORM TV DRAMA - PPTECONOMIC CONTEXT - LONG FORM TV DRAMA - PPT
ECONOMIC CONTEXT - LONG FORM TV DRAMA - PPTiammrhaywood
 
Field Attribute Index Feature in Odoo 17
Field Attribute Index Feature in Odoo 17Field Attribute Index Feature in Odoo 17
Field Attribute Index Feature in Odoo 17Celine George
 
How to do quick user assign in kanban in Odoo 17 ERP
How to do quick user assign in kanban in Odoo 17 ERPHow to do quick user assign in kanban in Odoo 17 ERP
How to do quick user assign in kanban in Odoo 17 ERPCeline George
 
MULTIDISCIPLINRY NATURE OF THE ENVIRONMENTAL STUDIES.pptx
MULTIDISCIPLINRY NATURE OF THE ENVIRONMENTAL STUDIES.pptxMULTIDISCIPLINRY NATURE OF THE ENVIRONMENTAL STUDIES.pptx
MULTIDISCIPLINRY NATURE OF THE ENVIRONMENTAL STUDIES.pptxAnupkumar Sharma
 
ENGLISH 7_Q4_LESSON 2_ Employing a Variety of Strategies for Effective Interp...
ENGLISH 7_Q4_LESSON 2_ Employing a Variety of Strategies for Effective Interp...ENGLISH 7_Q4_LESSON 2_ Employing a Variety of Strategies for Effective Interp...
ENGLISH 7_Q4_LESSON 2_ Employing a Variety of Strategies for Effective Interp...JhezDiaz1
 

Recently uploaded (20)

Hierarchy of management that covers different levels of management
Hierarchy of management that covers different levels of managementHierarchy of management that covers different levels of management
Hierarchy of management that covers different levels of management
 
Framing an Appropriate Research Question 6b9b26d93da94caf993c038d9efcdedb.pdf
Framing an Appropriate Research Question 6b9b26d93da94caf993c038d9efcdedb.pdfFraming an Appropriate Research Question 6b9b26d93da94caf993c038d9efcdedb.pdf
Framing an Appropriate Research Question 6b9b26d93da94caf993c038d9efcdedb.pdf
 
Gas measurement O2,Co2,& ph) 04/2024.pptx
Gas measurement O2,Co2,& ph) 04/2024.pptxGas measurement O2,Co2,& ph) 04/2024.pptx
Gas measurement O2,Co2,& ph) 04/2024.pptx
 
Employee wellbeing at the workplace.pptx
Employee wellbeing at the workplace.pptxEmployee wellbeing at the workplace.pptx
Employee wellbeing at the workplace.pptx
 
Like-prefer-love -hate+verb+ing & silent letters & citizenship text.pdf
Like-prefer-love -hate+verb+ing & silent letters & citizenship text.pdfLike-prefer-love -hate+verb+ing & silent letters & citizenship text.pdf
Like-prefer-love -hate+verb+ing & silent letters & citizenship text.pdf
 
Introduction to AI in Higher Education_draft.pptx
Introduction to AI in Higher Education_draft.pptxIntroduction to AI in Higher Education_draft.pptx
Introduction to AI in Higher Education_draft.pptx
 
Rapple "Scholarly Communications and the Sustainable Development Goals"
Rapple "Scholarly Communications and the Sustainable Development Goals"Rapple "Scholarly Communications and the Sustainable Development Goals"
Rapple "Scholarly Communications and the Sustainable Development Goals"
 
How to Configure Email Server in Odoo 17
How to Configure Email Server in Odoo 17How to Configure Email Server in Odoo 17
How to Configure Email Server in Odoo 17
 
Difference Between Search & Browse Methods in Odoo 17
Difference Between Search & Browse Methods in Odoo 17Difference Between Search & Browse Methods in Odoo 17
Difference Between Search & Browse Methods in Odoo 17
 
Grade 9 Q4-MELC1-Active and Passive Voice.pptx
Grade 9 Q4-MELC1-Active and Passive Voice.pptxGrade 9 Q4-MELC1-Active and Passive Voice.pptx
Grade 9 Q4-MELC1-Active and Passive Voice.pptx
 
EPANDING THE CONTENT OF AN OUTLINE using notes.pptx
EPANDING THE CONTENT OF AN OUTLINE using notes.pptxEPANDING THE CONTENT OF AN OUTLINE using notes.pptx
EPANDING THE CONTENT OF AN OUTLINE using notes.pptx
 
Proudly South Africa powerpoint Thorisha.pptx
Proudly South Africa powerpoint Thorisha.pptxProudly South Africa powerpoint Thorisha.pptx
Proudly South Africa powerpoint Thorisha.pptx
 
ECONOMIC CONTEXT - PAPER 1 Q3: NEWSPAPERS.pptx
ECONOMIC CONTEXT - PAPER 1 Q3: NEWSPAPERS.pptxECONOMIC CONTEXT - PAPER 1 Q3: NEWSPAPERS.pptx
ECONOMIC CONTEXT - PAPER 1 Q3: NEWSPAPERS.pptx
 
Judging the Relevance and worth of ideas part 2.pptx
Judging the Relevance  and worth of ideas part 2.pptxJudging the Relevance  and worth of ideas part 2.pptx
Judging the Relevance and worth of ideas part 2.pptx
 
Earth Day Presentation wow hello nice great
Earth Day Presentation wow hello nice greatEarth Day Presentation wow hello nice great
Earth Day Presentation wow hello nice great
 
ECONOMIC CONTEXT - LONG FORM TV DRAMA - PPT
ECONOMIC CONTEXT - LONG FORM TV DRAMA - PPTECONOMIC CONTEXT - LONG FORM TV DRAMA - PPT
ECONOMIC CONTEXT - LONG FORM TV DRAMA - PPT
 
Field Attribute Index Feature in Odoo 17
Field Attribute Index Feature in Odoo 17Field Attribute Index Feature in Odoo 17
Field Attribute Index Feature in Odoo 17
 
How to do quick user assign in kanban in Odoo 17 ERP
How to do quick user assign in kanban in Odoo 17 ERPHow to do quick user assign in kanban in Odoo 17 ERP
How to do quick user assign in kanban in Odoo 17 ERP
 
MULTIDISCIPLINRY NATURE OF THE ENVIRONMENTAL STUDIES.pptx
MULTIDISCIPLINRY NATURE OF THE ENVIRONMENTAL STUDIES.pptxMULTIDISCIPLINRY NATURE OF THE ENVIRONMENTAL STUDIES.pptx
MULTIDISCIPLINRY NATURE OF THE ENVIRONMENTAL STUDIES.pptx
 
ENGLISH 7_Q4_LESSON 2_ Employing a Variety of Strategies for Effective Interp...
ENGLISH 7_Q4_LESSON 2_ Employing a Variety of Strategies for Effective Interp...ENGLISH 7_Q4_LESSON 2_ Employing a Variety of Strategies for Effective Interp...
ENGLISH 7_Q4_LESSON 2_ Employing a Variety of Strategies for Effective Interp...
 

Prof ed set b questionati and answer

  • 1. PROFESSIONAL EDUCATION MULTIPLE CHOICE 1. You are very much interested in a quality professional development program for teachers. What characteristic should you look for? a. Prescribe by top educational teachers b. Dependent on the availability of funds c. Required for renewal of professional license d. Responsive to identified teacher’s needs. 2. To ensure high standards of teachers’ personal and professional development, which of the following measures must be implemented? I. A school head plans the professional development of his/her teachers. II. Every teacher formulates his/her own professional development plan III. The implementation of what is learned in a training must be monitored. a. I only II and III b. I and III d. II only 3. As a community leader, which of the following should a teacher NOT do? a. Support effort of the community to improve their status in life. b. Make herself aloof to ensure that her decisions will not be influenced by the community politics. c. Solicit donation from philanthropists in the community. d. Play an active part in the activities of the community. 4. In a highly pluralistic society, what type of learning environment is the responsibility of the teacher? I. Safe II. Gender-biased III. Secure a. I and II c. II only b. I, II and III d. I and III 5. A teacher is said to be “trustee of the cultural and educational heritage of the nation and is under obligation to transmit to learners such heritage”. Which practice makes the teacher fulfill such obligation? a. Use interactive teaching strategies. b. Use the latest educational technology. c. Observe continuing professional education d. As a class, study the life of Filipino heroes. 6. Which actions show that political factors affect schools as agents of change? I. The strengthening of the teaching of English in Philippines school. II. The introduction of mandated subjects such as Rizal in the curriculum III. The practice of mainstreaming IV. The turnover of day care centers for DSWD to DepEd for supervision. a. I and III c. II and III b. I and II d. II and IV 7. For more efficient and effective management of school as agents of change, one proposal is for the DepEd to cluster remote stand-alone schools under one lead school head. Which factor has the strongest influence on this proposal? a. Psychological c. Geographical b. Historical d. Social 8. What does the acronym EFA imply for schools? a. The acceptance of exclusive schools for boys and for girls. b. The stress on the superiority of formal education over that of alternative learning system. c. Practice of inclusive education d. The concentration on formal education system 9. The wide acceptance of “bottom up” management style has influenced schools to practice which management practice? a. Exclusion of politicians from the pool of guest speakers during graduation exercises. b. Prescription of what ought to be done from the Center Office. c. Involvement of students, parents, teachers, and community in school planning d. Allowing schools to do what they think is best 10. Large class size in congested cities is a common problem in our public schools. Which measure/s have schools taken to offset the effects of large class? I. The deployment of more teachers II. The implementation of 1:1 pupil textbook ratio III. The conduct of morning and afternoon sessions a. I, II and III c. III only b. I and II d. II only 11. The failure of independent study with most Filipino students may be attributed to students’ a. unpreparedness fro schooling b. ambivalence c. high degree of independence d. high degree of independence on authority 12. The schooling incidents in school campuses abroad have made school to rethink the curriculum. Which is believed to counteract such incidents and so is being introduced in schools? I. Inclusion of socio-emotional teaching II. The emphasis on the concept of competition against self and not against others III. Focus on academic achievement and productivity a. I and III c. I and II b. II and III d. I, II and III 13. Widespread abuse of Mother Earth prompted schools to teach sustainable development. which one does this prove about schools? a. The curricula of schools are centered on Mother Earth. b. Schools can easily integrate sustained development in their curriculum. c. Sustained development cannot be effectively taught in the classroom. d. Environment factors influence the school as an agent of change. 14. A father tells his daughter “You are a woman. You are meant for the home and so for you, going to school is not necessary.” Is the father CORRECT? a. It depends on the place when the daughter and the father live. b. Yes, women are meant to be a mother only. c. No. today women can take on the jobs of men. d. No, there is gender equality in education. 15. Is there a legal basis for increasing the teacher’s starting salary to P18,000 an months?
  • 2. a. No, it is a gift to teachers from Congress b. Yes, R.A 7836 c. No, it is simply an act of benevolence from President GMA d. Yes, the Phil. Constitution 16. As provided for the Educational Act of 1982, how are the institutions of learning encourage to set higher standards of equality over the minimum standards required for state recognition? a. Granting of Special Permit b. Academic freedom c. Continuing Professional Education d. Voluntary accreditation 17. Despite of opposition from some school official, DepEd has continuously enforced the “no collection of fees” policy during enrolment period in public schools. In this policy in accordance with EFA goals? a. No, it violates the mandate of equality education b. Yes, it somewhat eliminates gender disparities c. Yes, it supports equitable access to basic education d. No. it does not support parent of adult education 18. “Specialization is knowing more and more about less and less”. Hence, it is better to be a generalist, claims Teacher F. Which Philosophy does Teacher F. subscribe to? a. Existentialism c. Essentialism b. Perennialism d. Progressivism 19. Mencius believed that all people are born good. This thought on the innate goodness of people makes it easier to our pupils. a. teach c. like b. Respect d. motivate 20. The specialization requires of every professional teacher for him/her to be competent is in line with which pillar of learning? a. Learning to know b. Learning to be c. Learning to live together d. Learning to do 21. Which pillar of learning is aimed at the wholistic development of man and his complete fulfillment? a. Learning to be b. Learning to know c. Learning to live together d. Learning to do 22. Material development at the expense of human development points to the need to do more in school. a. “Learning to do” b. “Learning to know” c. “Learning to live together” d. “Learning to be” 23. A student complains to you about his failing grade. When you recomputed you found out that you committed an error in this grade computation. Your decision is not accept the erroneous computation before the student and so leave the failing grade as is for fear that you may lose credibility. Is this morally right? a. No, the reason for not accepting the error before the students in flimsy. b. No, the end does not justify the means c. Yes, the end justifies the means d. Yes, as a teacher you must maintain your credibility 24. Which violate(s) the principle of respect? I. Teacher A tells her students that what Teacher B taught is wrong. II. To retaliate, Teacher B advises students not to enroll in Teacher A’s class. III. Teacher C secretly gives way to a special favor (e.g. add 2 points to grade) requested by student A who is vying for honors. a. II and III c. I and II b. I, II and III d. I and III 25. Which is/are in accordance with the principle of pedagogical competence? I. Communication of objectives of the course to students II. Awareness of alternative instruction strategies III. Selection of appropriate methods of instruction a. I and III c. III only b. I, II and III d. II and III 26. With a death threat over his head, Teacher D is directed to pass an undeserving student. Which will a utilitarianist do? a. Pass the student, why suffer the threat? b. Don’t pass him; live by your principle of justice. You will get reward, if not in this life, in the next. c. Pass the student. That will be of use to the student, his parents and you. d. Don’t pass him. You surely will not like someone to give you a death threat in other to pass. 27. Teacher A knows of the illegal activities of a neighbor but keeps quiet in order not to be involved in any investigation. Which foundational principle of morality does Teacher A fail to apply? a. The end does not justify the means b. The end justifies the means c. Always do what is right d. Between two evils, do the lesser evil 28. To earn units for promotion, a teacher pays her fee but does not attend class at all. Does this constitute professional growth? a. Not immediately but yes after promotion b. It depends on the school she is enrolled in c. No, it is simply earning MA units for promotion d. Yes, just enrolling in an MA program is already professional growth 29. If a teacher asks more higher-order questions, he has to ask more questions. a. fact c. convergent b. close d. concept 30. Misdemeanor has a “ripple effect”. This implies that as a classroom manager, a teacher a. reinforces positive behavior b. responds to misbehavior promptly c. is consistent in her classroom management practice d. count 1 to 10 before she deals with a misbehaving student 31. Based on Edgar Dale’s “Cone of Experience”, which activity is farthest from the real thing? a. Watching demo c. Video disc
  • 3. b. Attending exhibit d. Viewing images 32. The students of Teacher Y scan an electronic encyclopedia, view a film on subject, or look at related topics at the touch of a button right there in the classroom. Which device/s des teacher Y’s class have? a. Teaching machines b. CD c. Video disc d. Videotaped lesson 33. Which is an INAPPROPIATE way to manage off-task behavior? a. Redirect a child’s attention to task and check his progress to make sure he is continuing work b. Make eye contact to communicate what you wish to communicate c. Move closer to the child to make him feel your presence d. Stop your class activity to correct a child who is no longer on task 34. To be an effective classroom manager, a teacher must be friendly but must at the same time be . a. confident c. analytical b. businesslike d. buddy-buddy 35. Which software is needed when one wants to perform automatic calculations on numerical data? a. Database b. Spreadsheet Program c. Microsoft Word d. Microsoft Powerpoint 36. Which of the following questions must be considered in evaluating teacher-made materials? a. In the material new? b. Does the material simulate individualism? c. Is the material expensive? 37. Kounin claims that “with-it-ness” is one of the characteristics of an effective classroom manager. What is one sign of “with- it-ness”? a. Giving attention to students who are having difficulty with school work b. Seeing only a portion of the class but intensively c. Knowing where instructional materials are kept d. Aware of what’s happening in all parts of the classroom 38. Which of these is one of the ways by which the internet enables people to browse documents connected by hypertext links? a. URL b. Browser c. Welcome page d. World Wide Web 39. Which characteristics must be primary considered as a choice of instructional aids? a. Stimulate and maintain students interests b. Suited to the lesson objectives c. Updated and relevant to Filipino setting d. New and skillfully made 40. You can exhibit referent power on the first day of school by a. telling them the importance of good grades b. giving your students a sense of belongingness and acceptance c. making them feel you know what you are taking about d. reminding your students your authority over them again and again 41. I would like to use a model to emphasize particular part. Which of these would be MOST appropriate? a. Regalia c. Stimulation b. Audio recording d. Mock up 42. What must a teacher do to ensure orderly transitions between activities? a. Allow time for the students to socialize in between activities b. Have the materials ready at the start of the activity c. Assign fewer exercise to fill the allotted time d. Wait for students who lag behind 43. The task of setting up routine activities for effective classroom management is a task that a teacher should undertake a. as soon as the students have adjusted to their schedules b. on the very first day of school c. every day at the start of the session d. every homeroom day 44. Teacher S uses the low-profile classroom control technique most of the time. what does this imply? a. She is reactive in her disciplinary orientation b. She manages pupils personalities c. She reacts severely to a misbehaving student d. She stops misbehaving without disrupting lesson flow 45. When teacher tries to elicit clarification on a student response or solicits additional information, which of these should be use? a. Directing c. Structuring b. Probing d. Cross examining 46. Which priority criterion should guide a teacher in the choice of instructional devices? a. Novelty c. Attractiveness b. Cost d. Appropriateness 47. Which learning activity is MOST appropriate if a teacher’s focus is attitudinal change? a. Fieldtrip c. Role play b. Exhibit d. Game 48. Teacher H strives to draw participation of every student into her classroom discussion. Which of these student needs is she trying to address? The need to . a. shoe one’s oral abilities to the rest of the class b. feel significant and be part of a group c. get everything and be part of a group d. be creative 49. Instead of teacher giving this comment a student response. “You’re on the wrong track!”, what should be teacher do? a. Change the question to an easier one b. Redirect the question by calling another student to recite
  • 4. c. Probe to redirect the response into a more productive area d. Pause, ask the question, lecture, then ask the question again 50. If curriculum is designed following the traditional approach, which feature(s) apply(ies)? I. The aims of the curriculum are set by professionals and experts II. Interested groups (teachers, students, communities) are assumed to agree with the aims of the curriculum III. Consensus building in not necessary a. III only c. I, II b. I, II, III d. I, III 51. I want my student to develop the ability to look at a problem from various perspectives. Which approach will be MOST fit? a. Behaviorist approach b. Computer-based Education c. Modular approach d. Cognitive approach 52. One’s approach to teaching is influenced by Howard Gardner’s MI Theory. What is he/she challenged to do? I. To come up with 9 different ways of approaching lesson to cater to the 9 multiple intelligence II. To develop all student’s skill in all nine intelligences III. To provide worthwhile activities that acknowledge individual difference in children a. I, II and III c. II only b. II, III d. III only 53. If my approach to my lesson is behaviorist, what features will dominate my lesson? I. Copying notes III. Lecturing II. Reasoning IV. Demonstration a. III, IV c. I, II, III, IV b. I, III, IV d. II, III, IV 54. You practice inclusive education. Which of these applies to you? I. You accept every student as full and valued member of the class and school community II. Your special attention is on learners with specific learning or social needs III. Your address the needs of the class as a whole within the context of the learners with specific learning or social needs a. II only c. I only b. I and II d. I and III 55. School curriculum reflects the world’s economic and political integration and industrialization. What does this point in curriculum development? a. The trend towards the classical approach to curriculum development b. The trend toward the globalization and localization c. The trend toward participatory curriculum development d. The shift in the paradigm of curriculum development from a process-oriented to a product-oriented one 56. You choose cooperative learning as a teaching approach. What thought is impressed on your students? a. Interaction is a must, but not necessarily face to-face interaction b. Student’s success depends on the success of the group c. Student’s individuality evaluate how effectively their group worked. d. The accountability for learning is on the group not on the individual 57. What principle is violated by overusing the chalkboard, as though it is the only education technology available? a. Isolated use c. Variety b. Flexibility d. Uniformity 58. Which statement applies a CORRECTLY to Edgar Dale’s ”Cone of Experience” a. The farther you are from the base, the more direct the learning experience becomes. b. The farther you are from the bottom, the more direct the learning experience becomes. c. The closer you are from the base, the more indirect the learning experience becomes d. The closer you are from the base, the more direct the learning experience becomes 59. “When more senses are stimulated, teaching and learning become more effective.” What is an application of this principle? a. Appeal to student’s sense of imagination b. Use multisensory aids c. Make your students touch the instructional material d. Use audiovisual aids because the eyes and the eras are the most important senses in learning. 60. Which is a classroom application of the theory of “operant conditioning”? a. Help student see the connectedness of facts, concepts, and principles b. Create a classroom atmosphere that elicits relaxation c. Reinforce a good behavior to increase the likelihood that the learner will repeat the response d. Make students learn by operating manipulatives Read the following teacher-student situation 61. TEACHER: Why is the process called photosynthesis? STUDENT: I don’t know Which questioning technique should be the teacher use? a. Clarification c. Prompting b. Multiple response d. Concept review 62. Here is the test item. “From the data presented in the table, from generalizations that are supported by the data”. Under what type of question does this item fall? a. Convergent c. Application b. Evaluative d. Divergent 63. I want to teach concepts, patterns and abstractions. Which method will be MOST appropriate? a. Discovery c. Direct instruction b. Indirect instruction d. Problem solving 64. Teacher A teaches English as a Second Language. She uses vocabulary cards, fill-in-the-blanks sentences, dialogues, dictation and writing exercises in teaching a lesson about grocery shopping. Based on this information, which of the following is a valid conclusion? a. The teacher wants to make her teaching easier by having les talk b. The teacher emphasizing reading and writing skills c. The teacher is teaching in a variety of ways because not all students learn in the same manner d. The teacher is applying Bloom’s hierarchy of cognitive learning 65. Teacher A an experienced teacher, does daily review of past lessons in order to
  • 5. a. introduce a new lesson b. reflect on how he presented the previous lessons c. provide his pupils with a sense of continuity d. determine who among his pupils are studying 66. I combined several subject areas in order to focus on a single concept for interdisciplinary teaching. Which strategy did I use? a. Reading-writing activity b. Thematic introduction c. Unit method d. Problem-centered learning 67. To teach the democratic process to the pupils, Biag Elementary School decided that the election of class officers shall be patterned after local elections. There are qualifications set for candidates, limited period for campaign and rules for posting campaign materials, etc. Which of the following did the school use? a. Symposium c. Pole playing b. Simulation d. Philips 66 68. Which are effective methods in teaching student critical reading skills? I. Interpret editorial II. Read and interpret three different movie reviews III. Read a position paper and deduce underlying assumptions of the position papers a. II and III c. I and II b. I and III d. I, II and III 69. Here is a test item “The improvement of basic education should be the top priority of the Philippine government. Defend or refute this position”. Under what type of question does this test item fall? a. Low-level c. Analysis b. Evaluative d. Convergent 70. When I teach, I often engage in brainstorming. Which do I avoid? a. Break down barriers b. Increase creativity c. Generate many ideas d. Selectively involves pupils 71. Teacher S teaches a lesson in which students must recognize that ¼ is the same 0.25. They use this relationship to determine that 0.15 and 0.20 are slightly less than ¼. Which of the following concept/s is/are being taught? a. Numeration skills b. Place value of decimals c. Numeration skills of decimals and relationships between fractions and decimals d. Relationship between fraction and decimals 72. What is the best way to develop math concept? a. Solving problems using multiple approaches b. Solving problems by looking for correct answer c. Learning math as applied to situations, such as being a tool of science d. solving problems by applying learned formulas 73. After the reading of a selection in the class, which of these activities can enhance students creativity. I. Reader’s theater II. Reading aloud III. Silent reading a. I and II c. I only b. II only d. III only 74. Teacher C, a Reading teacher, advised he class to “read between the lines”. What does she want her pupils to do? a. Make an educated guess b. Determine what is meant by what is stated c. Apply the information read d. Describe the characters in the story 75. To nurture students’ creativity, which activity should a teacher AVOID? a. Ask “hat if…” questions b. Ask divergent thinking questions c. Emphasize the need to give right answers d. Be open to “out-of-this-world” ideas 76. Teacher R wants to develop his student’s creativity. Which type of questions will be MOST appropriate? a. Synthesis questions b. Fact questions c. “What if….” questions d. Analysis questions 77. In my attempt to develop creative thinking skills, I want to test fluency of ideas. Which activity for my students will be MOST appropriate? a. Solve this math problem b. List animals covered with hair in 1 minute c. Solve this puzzle d. Compare pictures 1 and 2. Where are the differences? 78. You want your students to answer the questions at the end of a reading lesson. “What did I learn did?”,”What still puzzle me?”, “What did I enjoy, hate accomplish in the class today”?,”How did I learn from the lesson?”.Which of the following are you asking them to do? a. Work on an assignment b. Make journal entry c. Work on a drill d. Apply what they learned 79. After reading an essay. Teacher B wants to help sharpen her students’ ability to interpret. Which of these activities will be most appropriate? a. Drawing conclusions b. Making inferences c. Getting the main idea d. Listing facts separately from opinion Read the following then answer the question
  • 6. 80. A man and his son are driving in a car. The car crashes into a tree, killing the father and seriously injury his son. At the hospital, the boy needs to have surgery. Looking at the boy, the doctor says (telling the truth),”I cannot operate on him. He is my son. How can this be? ASWER: The doctor is the boy’s mother. The above brain twister helps develop critical reading skills. Which activity was used? a. Comparing b. Classifying c. Inferring meaning d. Looking for cause and effect 81. Research says that mastery experiences increase confidence and willingness to try similar or more challenging tasks as reading. What does this imply for children’s reading performance? a. Children who have not mastered the basic skills are more likely to be motivated to read in order to gain mastery over basic skills. b. Children who have mastered basic skills are more likely to be less motivated to read because they get fed up with too much reading. c. Children who have a high sense of self-confidence are not necessarily those who can read d. Children who have gained mastery over basic skills are more motivated to read 82. The value that students put on reading is critical to their success. In what way/s can teachers inculcate his value for reading? I. Sharing the excitement of read-aloud II. Showing their passion for reading III. Being rewarded to demonstrate the value of reading a. II and III c. I, II and III b. I and II d. II only 83. Bruner’s theory on intellectual development moves from enactive to iconic and symbolic stages. Applying Bruner’s theory. How would you teach? a. Be interactive in approach b. Begin with the abstract c. Begin with the concrete d. do direct instruction 84. A person who has painful experiences at the dentist’s office may become fearful at the mere sight of the dentist’s office building. Which theory can explain this? a. Generalization b. Operant Conditioning c. Attribution theory d. Classical conditioning 85. Which is/are the basic assumption/s of behaviorists? I. The mind of newborn child is a blank state. II. all behaviors are determined by environmental events III. The child has a certain degree of freedom not to allow himself to be shaped by his environment. a. III only c. II only b. I and II d. I and II 86. If a student is encourage to develop himself to the fullest and must satisfy his hierarchy of needs, the highest needs to satisfy according to Maslow is . a. psychological need c. belongingness b. self-actualization d. safety needs 87. In a Social studies class. Teacher I presents a morally ambiguous situation and asks student what they would do. On whose theory is Teacher I’s technique based? a. Bandura c. Kohberg b. Piaget d. Bruner 88. Teacher F is convinced that whatever a student performs a desired behavior, provide reinforcement and soon the student learns to perform the behavior on his own. On which principle is Teacher F’s conviction based? a. Environmentalism c. Cognitivism b. Behaviorism d. Constructivism 89. Bandura’s social learning theory, states that children often imitate those who I. have substantial influence over their lives II. belong their peer group III. belong to other races IV. are successful and seem admired a. IV only c. I and II b. I and IV d. II and IV 90. According to Erikson, what years are critical for the development of self-confidence? a. High school years b. College years c. Preschool years d. Elementary school years 91. Which of the following does NOT describe the development of children aged 11 to 13? a. They exhibit increased objectivity in thinking b. They shift from impulsivity to adaptive ability c. Sex difference in IQ become more evident d. They show abstract thinking and judgment 92. Teacher H begins a lesson on tumbling, demonstrating front and back somersaults in slow motion and physically guiding his students through the correct movements. As his students become more skillful, he stands back from the man and gives verbal feedback about how to improve. With Vygotsky’s theory in mind, what did Teacher H do? a. Apprenticeship b. Guided participation c. Peer interaction d. Scaffolding 93. What does Gagne’s hierarchy theory propose for effective instruction? a. Be concerned with the socio-emotional climate in the classroom b. Teach beginning with the concrete c. Sequence instruction d. Reward good behavior 94. William Glasser’s control theory states that behavior in inspired by what satisfies a person’s want at any given time. What then must a teacher do to motivate students to learn? a. Make teaching-leaning interactive
  • 7. b. Avoid giving assignments c. Organize a curriculum in a spiral manner d. Make schoolwork relevant to students’ basic human needs. 95. Soc exhibit fear response to freely roaming dogs but does not show fear when a dog is on a leash or confined to a pen. Which conditioning process is illustrated? a. extinction c. acquisition b. generalization d. discrimination 96. Based on Freud’s theory, which operate/s when a student strikes a classmates at the height of anger? a. Ego c. Id and Ego interact b. Id d. Superego 97. Bernadette enjoyed the roller coaster when he and her family went to Enchanted Kingdom. The mere sight of a roller coaster gets her excited. Which theory explains Bernadette’s behavior? a. Operant conditioning b. Social learning theory c. Attribution theory d. Pavlovian conditioning 98. According to Frued, with which should one be concerned if he/she has to develop in the students a correct sense of right and wrong? I. Super-ego II. Ego III. Id a. I and II c. I b. II d. III 99. When small children call animals “dog”, what process is illustrated on Piaget’s cognitive development theory? a. reversion c. accommodation b. assimilation d. conservation 100. Researchers found that when a child is engaged in a learning experience a number of areas of the brain are simultaneously activated. Which of the following is/are implication/s of this research finding? I. Make use of field trips, guest speakers II. Do multicultural units of study III. Stick to the “left brain and right brain” approach a. I and III c. I and II b. I only d. II only 101. Which appropriate teaching practice flows from this research finding on the brain: “The brain’s emotional center is tied into its ability to learn”. a. Establish the discipline of being judgmental in attitude b. Come up with highly competitive games where winners will feel happy c. Tell the students to participate in class activities or else won’t receive plus points in class recitation d. Create a learning environment that encourages students to explore their feeling and ideas freely 102. Research on Piagetian tasks indicates that thinking becomes more logical and abstract as children reach the formal operations stage. What is an educational implication of this finding? a. Engage children in analogical reasoning as early as preschool to train them for higher order thinking skills (HOTS) b. Learners who are not capable of logical reasoning from ages 8 to 11 lag behind in their cognitive development c. Let children be children d. Expect hypothetical reasoning for learners between 12 to 15 years of age 103. Research says: “People tend to attribute their successes to internal causes and their failures to external causes.”Based on this finding, what should be taught to students for them to be genuinely motivated to succeed? a. Tell them the research finding when applied will make them genuinely motivated b. Convince them that genuine motivation is the only factor that matters for a person to succeed c. Make them realize that failure is a part of life d. Make them realize that both success and failure are more a function of internal causes. 104. Which characterize/s a learning environment that promotes fairness among learners of various cultures, family background and gender? I. Inclusive II. Exclusive III. Gender-sensitive a. I only c. I and III b. III only d. II and III 105. Which of the following steps should be completed first in planning an achievement test? a. Define the instructional objective b. Set up a table of specialization c. Select the types of test items to use d. Decide on the length of the test 106. The computed r fro scores in Math and Science in 0.92. What does this mean? a. Math score is positive related to Science score b. The higher the Math score, the lower the Science score c. Math score is not in any way related to Science score d. Science score is slightly related to math score 107. Which types of test is most appropriate if Teacher Y wants to measure student’s ability to organize thoughts and ideas? a. Short answer type of test b. Extended response essay c. Modified alternative response d. Limited response essay 108. With assessment of affective learning in mind, which does NOT belong to the group? a. Cloze test c. Reflective writing b. Moral dilemma d. Diary entry 109. I want to test student’s synthesizing skills. Which has the highest diagnostic value? a. multiple choice test c. Essay test b. Performance test d. Completion test 110. Why is this test item poor? is an example of a leafy vegetable.
  • 8. I. The test item does not pose a problem to the examinee II. There are variety of possible correct answer to this item III. the language used in the question is not precise IV. The blank is near the beginning of a sentence A. I and III B. II and IV C. I and IV D. I and II 111. What makes the multiple choice type of test poor? a. The options are not grammatically connected to the stem b. The stem fails to present a problem c. There are grammatical clues d. The options are not parallel 112. If a teacher wants to measure her students’ ability to discriminate, which of these is an appropriate type of test item as implied by the direction? a. “Outline the Chapter on The Cell”. b. “Summarize the lesson yesterday”. c. “Group the following items according to shape”. d. “State a set of principle that can explain the following events”. 113. A test item has a difficult index of 0.89 and a discrimination index of 0.44. What should the teacher do? a. Reject the item b. Retain the item c. Make it a bonus item d. Make it a bonus item and reject it 114. Which form of assessment is consistent with the saying “The proof of the pudding is in the eating”. a. Contrived c. Traditional b. Authentic d. Indirect 115. What is WRONG with this item? a. Item is overly specific b. Content is trivial c. Test item is option-based d. There is a cue to the right answer 116. Student’s score were as follows: 82, 83, 84, 86, 88, 84, 83, 85. The score 86 is the. a. mode c. median b. average d. mean 117. Which text form would you choose if you want to have a valid and reliable test based on the table below? Test Form Validity Index Reliability Index A .47 .68 B .87 .57 C .20 .86 D .40 .41 E .63 .07 a. A only c. A and D b. B only d. B and E 118. A mathematicians test was given to all Grade V pupils to determine the contestants for the Math Quiz Bee. Which statistical measure should be used to identify the top 15? a. Mean percentage score b. Quartile Deviation c. Percentile Rank d. Percentage Score 119. Nellie’s score is within x±1 SD. To which of the following groups does she belong? a. Below average b. Average c. Needs Improvement d. Above average 120. Use the inbox below to answer the question that follows: Percentage Grades for Final Examination What follows is a multiple choice type of test. Some test items . a. are too difficult b. are objective c. are poorly constructed d. have multiple defensible answers Who is best admired for outstanding contribution to world peace? a. Kissinger c. Kennedy b. Clinton d. Mother Teresa
  • 9. 40 70 80 90 100 Which of the following statement is TRUE about the plot of grades above? a. The median is a score of 80 and the range is 60. b. The median is a score of 70 and the range is 60. c. The median is a score of 80 and the range is 20. d. The median is a score of 70 and the range is 20. 121. Which can be said of Arielle who obtained a score of 75 out of 100 items in a Grammar objective test? a. She performed better than 25% of her classmates b. She answered 75 items in the test correctly c. Her rating is 75 d. She answered 75% of the test items correctly 122. The criterion of success in Teacher D’s objective is that “the pupils must be able to spell 90% of the words correctly”. Ana and 24 others in the class spelled only 40 out of 50 words correctly while the rest scored 45 and above. This means that Teacher D a. attained her lesson objective b. did not attain her lesson objective because of the pupils’ lack of attention c. failed to attain her lesson objective as far as the 25 pupils are concerned d. attained her lesson objective because of her effective spelling drill 123. If the scores of your test follow a negatively skewed score distribution, what should you do? Find out ? a. why your items were easy b. why most of the scores are high c. why most of the scores are low d. why some pupils scored high 124. Principal A is talking about “grading on the curve” in a faculty meeting. What does this expression refer to? a. A student mark compares his achievement to his effort. b. A student’s grade or mark depends on how his achievement compares with the achievement of other students in a class. c. A student’s grade determines whether or not a student attains a defined standard of achievement d. A student mark tells how closely he is achieving to his potential. 125. Which tests determine whether students accept responsibility for their own behavior or pass on responsibility for their own behavior to other people? a. Thematic tests b. Sentence-completion tests c. Stylistic test d. Locus-of-control tests 126. Which of the one weakness of self-supporting personality checklists? a. Many personality measures have built-in lie scales b. They lack stability c. They may not get true information because individuals can hide or disguise feelings d. They have poor internal consistency 127. Which of these can measure awareness of values? a. Sociogram b. Moral dilemmas c. Projective techniques d. Rating scales 128. Marking on a normative basis means that a. the normal distribution curve should be followed b. some should fall c. some get high marks d. the grading is based on a present criteria 129. Which process enhances the comparability of grades? a. Using a table specifications b. Determining the level of difficulty of the tests c. Giving more HOTS (higher order thinking skills) d. Constructing departmentalized examinations for each subject area. SITUATIONAL 130. Which response/s come/s from a behaviorist? a. #2 and #4 c. #3 and #4 b. #1 and #2 d. #1 and #3 131. On which educational philosophy is response #1 anchored? a. Existentialism c. Progressivism b. Essentialism d. Bahaviorism 132. If you learned toward a progressivist philosophy, with which response would you agree? a. #2 b. #3 c. #4 d. #1 Situation 1- In a faculty meeting, the principle told his teacher: “We need to improve our school performance in the National Achievement Test. What should we do? The teacher gave varied answers as follows: 1. Let’s give incentives and rewards to students who get a rating of 85% 2. Let’s teach them to accept complete responsibility for their performance 3. Let’s make the school environment conducive for learning 4. Let’s make use of the experimental methods of teaching Situation 2- One principle in the utilization of technology of the classroom is appropriateness of material or activity.
  • 10. 133. Teacher C wants his students to master the concept of social justice. Which series of activities will be most effective? a. Pretest-teaching-posttest b. Pretest-teaching-posttest-re-teaching for unlearned concepts-posttest c. Review-pretest-teaching-posttest d. Teaching-posttest 134. Teacher A likes to show how the launching of spaceships takes place. Which of the following materials available is most fit? a. Model c. Replica b. Mock-up d. Realia 135. Teacher B likes to concretize the abstract concepts of an atom. She came up with a concrete presentation of the atom by using wire and plastic balls. How would you classify Teacher B’s visual aids? a. Chart c. Model b. Replica d. Realia 136. The class was asked to share their insights about the poem. The ability to come up with a n insight stems from the ability to a. analyze the parts of a whole b. evaluate the worthiness of a thing c. relate and organize things and ideas d. comprehend the subject that is being studied 137. To ask the class any insight derived from the poem is based on the theory of a. realism c. conditioning b. behaviorism d. constructivism 138. On which assumption about the learner is Mr. Marquez’s act of asking the class to share their insight based? a. Learners are like empty receptacles waiting to be filled up b. Learners are meant to interact with one another c. Learners have multiple intelligence and varied learning styles d. Learners are producers of knowledge not only passive recipients of information 139. On which assumption/s is the principal’s action anchored? I. Students learn by personally constructing meaning of what is taught. II. Students are construct and reconstruct meaning based on experiences III. Students derive meaning from the meaning that the teacher gives a. II only c. I, II, and III b. I and II d. I only 140. Which materials will her teachers LEAST prefers? a. Controversial issues b. Open-ended topics c. Unquestionable laws d. Problem or cases 141. Which concept/s of the learner will Principal E NOT accept? I. “Empty vesse!” II. “Tabula rasa” III. Candle to be lighted a. III only c. II only b. I only d. I and II 142. How can you make the items homogeneous? a. Increase the number of items in Column B b. All items should be on polygons c. Remove the word triangle in items #1 and #2 in column A d. The word “gon” must be included in column B 143. What is the main defect of this matching test? a. the matching type is an imperfect type b. the items are NOT homogeneous c. the items quite easy d. an obvious pattern is followed in the answering 144. Which should be done to improve the matching type of test? a. Capitalize the items in Column A b. Items in Column A and B should be exchanged c. Drop #6 item in Column A Situation 3- After reading and paraphrasing Robert frost’s “Stopping by the Wood on a snowy Evening”. Mr. Sales asked the class to Situation 4- Principal E wants her teachers to apply constructivism in teaching Situation 5- Study the matching type of test then answer the 3 questions that follow: Column A Column B 1. equilateral triangle A. With 3 equal sides 2. right triangle B. With 5 equal sides 3. octagon C. Has 90- degree angle 4. pentagon D. Means many 5. heptagon E. with 7 sides 6. poly F. with 8 sides
  • 11. d. The item in Column A should be increased 145. Which of these is/are essential in constructing a scoring rubric? I. Description of criteria to serve as standard II. Clear descriptions of performance at each level III. Levels of achievement (mastery) IV. Rating scheme a. I, II, III c. I, II, III, IV b. I, II d. I only 146. Which statement is TRUE of the rubric? a. It is developmental b. It is analytical c. It is both holistic and developmental d. It is holistic 147. Which is TRUE of the scoring rubric? I. It describes criteria of levels of achievement II. It has a rating scheme III. It limit itself to 4 levels of achievement a. I and II b. I and III c. II and III d. I, II and III 148. The table shows that the test item analyzed . a. has a positive discrimination index b. has a negative discrimination index c. is extremely easy d. is extremely difficult 149. Based on the table, which is the most effective distracter? a. Option D b. Option A c. Option C d. Option B 150. Based on the table, which group got more correct answer? a. Upper group b. It cannot be determined c. Lower group d. Data are not sufficient to give an answer When you get right down to the root of the meaning of the word "succeed," you find that it simply means to follow through. Situation 6- Below the template for Scoring Rubric. 5-Demonstrate complete understanding of the problem. All requirements of task are included in response 4-Demonstrate considerable understanding of the problem. All requirements of task are included 3- Demonstrate partial understanding of the problem. Most requirements of task are included 2- Demonstrate little understanding of the problem. Many requirements of task are missing 1- Demonstrate no understanding of the problem 0-No response/task not attempted Situation 7- Study the table on item analysis for non-attractiveness and non-plausibility of distracters based on the results of a try-out test in Science. The letter marked with a asterisk is the correct answer. Item No. 1 A B C D E Upper 27% 10 4 1 1 0 Lower 27% 6 5 2 2 0
  • 12. Answer Key for prof ed 1 C 51 D 101 D 2 B 52 B 102 D 3 C 53 B 103 C 4 D 54 D 104 C 5 D 55 D 105 A 6 D 56 C 106 A 7 C 57 C 107 B 8 C 58 D 108 A 9 C 59 B 109 B 10 A 60 C 110 D 11 D 61 A 111 B 12 D 62 B 112 C 13 B 63 A 113 B 14 D 64 D 114 B 15 D 65 C 115 D 16 C 66 B 116 D 17 C 67 B 117 A 18 C 68 D 118 C 19 A 69 C 119 D 20 A 70 D 120 A 21 A 71 D 121 B 22 A 72 C 122 C 23 B 73 C 123 B 24 B 74 B 124 B 25 B 75 C 125 D 26 C 76 C 126 C 27 C 77 B 127 B 28 C 78 C 128 D 29 D 79 B 129 D 30 B 80 D 130 B 31 D 81 D 131 D 32 A 82 C 132 B 33 D 83 A 133 B 34 B 84 D 134 D 35 B 85 C 135 B 36 C 86 B 136 A 37 D 87 C 137 D 38 A 88 D 138 B 39 B 89 C 139 B 40 C 90 D 140 C 41 D 91 C 141 A 42 B 92 B 142 B 43 C 93 C 143 B 44 A 94 D 144 C 45 B 95 D 145 C 46 D 96 B 146 C 47 C 97 A 147 A 48 B 98 C 148 D 49 B 99 D 149 D 50 B 100 C 150 C